Prove that $D_{2n} = langle s, rs rangle$












0














Show that the subgroup of $D_{2n}$ generated by the set ${s,rs}$ is $D_{2n}$ itself.



Here is my attempt:



$langle s, rs rangle = D_{2n}$ if and only if $langle r, s rangle = langle s, rs rangle$, so it is sufficient to show that the latter equality holds. The containment $langle r, s rangle subseteq langle s, rs rangle$ holds: since $r, s in langle s, rs rangle$ (cleary this is true for $s$, and also $r=(rs)s$), every element in $langle r, s rangle$ is also an element of $langle s, rs rangle$. The containment $langle s, rs rangle subseteq langle r, s rangle$ is clear, because since $s,rs in langle r, s rangle$ every element in $langle s, rs rangle$ is also an element in $langle r, s rangle$. Therefore $langle r, s rangle = langle s, rs rangle = D_{2n}$.










share|cite|improve this question
























  • Your solution is perfect!
    – LucaMac
    Sep 6 '18 at 19:50












  • There's nothing special about $D_{2n}$ here besides that it has two generators. We can thus generalise it . . .
    – Shaun
    Dec 1 '18 at 17:45
















0














Show that the subgroup of $D_{2n}$ generated by the set ${s,rs}$ is $D_{2n}$ itself.



Here is my attempt:



$langle s, rs rangle = D_{2n}$ if and only if $langle r, s rangle = langle s, rs rangle$, so it is sufficient to show that the latter equality holds. The containment $langle r, s rangle subseteq langle s, rs rangle$ holds: since $r, s in langle s, rs rangle$ (cleary this is true for $s$, and also $r=(rs)s$), every element in $langle r, s rangle$ is also an element of $langle s, rs rangle$. The containment $langle s, rs rangle subseteq langle r, s rangle$ is clear, because since $s,rs in langle r, s rangle$ every element in $langle s, rs rangle$ is also an element in $langle r, s rangle$. Therefore $langle r, s rangle = langle s, rs rangle = D_{2n}$.










share|cite|improve this question
























  • Your solution is perfect!
    – LucaMac
    Sep 6 '18 at 19:50












  • There's nothing special about $D_{2n}$ here besides that it has two generators. We can thus generalise it . . .
    – Shaun
    Dec 1 '18 at 17:45














0












0








0







Show that the subgroup of $D_{2n}$ generated by the set ${s,rs}$ is $D_{2n}$ itself.



Here is my attempt:



$langle s, rs rangle = D_{2n}$ if and only if $langle r, s rangle = langle s, rs rangle$, so it is sufficient to show that the latter equality holds. The containment $langle r, s rangle subseteq langle s, rs rangle$ holds: since $r, s in langle s, rs rangle$ (cleary this is true for $s$, and also $r=(rs)s$), every element in $langle r, s rangle$ is also an element of $langle s, rs rangle$. The containment $langle s, rs rangle subseteq langle r, s rangle$ is clear, because since $s,rs in langle r, s rangle$ every element in $langle s, rs rangle$ is also an element in $langle r, s rangle$. Therefore $langle r, s rangle = langle s, rs rangle = D_{2n}$.










share|cite|improve this question















Show that the subgroup of $D_{2n}$ generated by the set ${s,rs}$ is $D_{2n}$ itself.



Here is my attempt:



$langle s, rs rangle = D_{2n}$ if and only if $langle r, s rangle = langle s, rs rangle$, so it is sufficient to show that the latter equality holds. The containment $langle r, s rangle subseteq langle s, rs rangle$ holds: since $r, s in langle s, rs rangle$ (cleary this is true for $s$, and also $r=(rs)s$), every element in $langle r, s rangle$ is also an element of $langle s, rs rangle$. The containment $langle s, rs rangle subseteq langle r, s rangle$ is clear, because since $s,rs in langle r, s rangle$ every element in $langle s, rs rangle$ is also an element in $langle r, s rangle$. Therefore $langle r, s rangle = langle s, rs rangle = D_{2n}$.







group-theory proof-verification dihedral-groups






share|cite|improve this question















share|cite|improve this question













share|cite|improve this question




share|cite|improve this question








edited Dec 1 '18 at 4:23









Shaun

8,820113681




8,820113681










asked Sep 6 '18 at 19:48









WesleyWesley

518313




518313












  • Your solution is perfect!
    – LucaMac
    Sep 6 '18 at 19:50












  • There's nothing special about $D_{2n}$ here besides that it has two generators. We can thus generalise it . . .
    – Shaun
    Dec 1 '18 at 17:45


















  • Your solution is perfect!
    – LucaMac
    Sep 6 '18 at 19:50












  • There's nothing special about $D_{2n}$ here besides that it has two generators. We can thus generalise it . . .
    – Shaun
    Dec 1 '18 at 17:45
















Your solution is perfect!
– LucaMac
Sep 6 '18 at 19:50






Your solution is perfect!
– LucaMac
Sep 6 '18 at 19:50














There's nothing special about $D_{2n}$ here besides that it has two generators. We can thus generalise it . . .
– Shaun
Dec 1 '18 at 17:45




There's nothing special about $D_{2n}$ here besides that it has two generators. We can thus generalise it . . .
– Shaun
Dec 1 '18 at 17:45










1 Answer
1






active

oldest

votes


















0














Your proof is sound, except that there is (what I suspect is) a typo. It should be $$r=(rs)s^{color{red}{-1}}.$$






share|cite|improve this answer





















    Your Answer





    StackExchange.ifUsing("editor", function () {
    return StackExchange.using("mathjaxEditing", function () {
    StackExchange.MarkdownEditor.creationCallbacks.add(function (editor, postfix) {
    StackExchange.mathjaxEditing.prepareWmdForMathJax(editor, postfix, [["$", "$"], ["\\(","\\)"]]);
    });
    });
    }, "mathjax-editing");

    StackExchange.ready(function() {
    var channelOptions = {
    tags: "".split(" "),
    id: "69"
    };
    initTagRenderer("".split(" "), "".split(" "), channelOptions);

    StackExchange.using("externalEditor", function() {
    // Have to fire editor after snippets, if snippets enabled
    if (StackExchange.settings.snippets.snippetsEnabled) {
    StackExchange.using("snippets", function() {
    createEditor();
    });
    }
    else {
    createEditor();
    }
    });

    function createEditor() {
    StackExchange.prepareEditor({
    heartbeatType: 'answer',
    autoActivateHeartbeat: false,
    convertImagesToLinks: true,
    noModals: true,
    showLowRepImageUploadWarning: true,
    reputationToPostImages: 10,
    bindNavPrevention: true,
    postfix: "",
    imageUploader: {
    brandingHtml: "Powered by u003ca class="icon-imgur-white" href="https://imgur.com/"u003eu003c/au003e",
    contentPolicyHtml: "User contributions licensed under u003ca href="https://creativecommons.org/licenses/by-sa/3.0/"u003ecc by-sa 3.0 with attribution requiredu003c/au003e u003ca href="https://stackoverflow.com/legal/content-policy"u003e(content policy)u003c/au003e",
    allowUrls: true
    },
    noCode: true, onDemand: true,
    discardSelector: ".discard-answer"
    ,immediatelyShowMarkdownHelp:true
    });


    }
    });














    draft saved

    draft discarded


















    StackExchange.ready(
    function () {
    StackExchange.openid.initPostLogin('.new-post-login', 'https%3a%2f%2fmath.stackexchange.com%2fquestions%2f2907899%2fprove-that-d-2n-langle-s-rs-rangle%23new-answer', 'question_page');
    }
    );

    Post as a guest















    Required, but never shown

























    1 Answer
    1






    active

    oldest

    votes








    1 Answer
    1






    active

    oldest

    votes









    active

    oldest

    votes






    active

    oldest

    votes









    0














    Your proof is sound, except that there is (what I suspect is) a typo. It should be $$r=(rs)s^{color{red}{-1}}.$$






    share|cite|improve this answer


























      0














      Your proof is sound, except that there is (what I suspect is) a typo. It should be $$r=(rs)s^{color{red}{-1}}.$$






      share|cite|improve this answer
























        0












        0








        0






        Your proof is sound, except that there is (what I suspect is) a typo. It should be $$r=(rs)s^{color{red}{-1}}.$$






        share|cite|improve this answer












        Your proof is sound, except that there is (what I suspect is) a typo. It should be $$r=(rs)s^{color{red}{-1}}.$$







        share|cite|improve this answer












        share|cite|improve this answer



        share|cite|improve this answer










        answered Nov 30 '18 at 19:17









        ShaunShaun

        8,820113681




        8,820113681






























            draft saved

            draft discarded




















































            Thanks for contributing an answer to Mathematics Stack Exchange!


            • Please be sure to answer the question. Provide details and share your research!

            But avoid



            • Asking for help, clarification, or responding to other answers.

            • Making statements based on opinion; back them up with references or personal experience.


            Use MathJax to format equations. MathJax reference.


            To learn more, see our tips on writing great answers.





            Some of your past answers have not been well-received, and you're in danger of being blocked from answering.


            Please pay close attention to the following guidance:


            • Please be sure to answer the question. Provide details and share your research!

            But avoid



            • Asking for help, clarification, or responding to other answers.

            • Making statements based on opinion; back them up with references or personal experience.


            To learn more, see our tips on writing great answers.




            draft saved


            draft discarded














            StackExchange.ready(
            function () {
            StackExchange.openid.initPostLogin('.new-post-login', 'https%3a%2f%2fmath.stackexchange.com%2fquestions%2f2907899%2fprove-that-d-2n-langle-s-rs-rangle%23new-answer', 'question_page');
            }
            );

            Post as a guest















            Required, but never shown





















































            Required, but never shown














            Required, but never shown












            Required, but never shown







            Required, but never shown

































            Required, but never shown














            Required, but never shown












            Required, but never shown







            Required, but never shown







            Popular posts from this blog

            How do I know what Microsoft account the skydrive app is syncing to?

            Grease: Live!

            When does type information flow backwards in C++?